LSAT and Law School Admissions Forum

Get expert LSAT preparation and law school admissions advice from PowerScore Test Preparation.

 Administrator
PowerScore Staff
  • PowerScore Staff
  • Posts: 8917
  • Joined: Feb 02, 2011
|
#47201
Please post your questions below!
 janietoto1029
  • Posts: 4
  • Joined: Aug 26, 2018
|
#58931
Hi,

I initially picked (B) but upon BR, I eliminated c, d, and e to be definitely false so zeroed in on (A), but I'm not sure how (A) can be true.

While the premise does say "Your intuitions tell you that the conclusion of a philosophical paradox is false", accepting this is not required as we can just not accept the conclusion to be true, based on the conclusion.

Any thoughts?
 Brook Miscoski
PowerScore Staff
  • PowerScore Staff
  • Posts: 418
  • Joined: Sep 13, 2018
|
#59196
Janie,

The word mash known as #20 is particularly baffling. However, there is a way to zero in on the correct answer reliably.

This question stem has no qualification--it's not asking what is most supported or most likely to be true. It simply asks what "must be true." That means the right choice is provable using the stimulus.

Separating into contenders and losers, I agree with you that c/d/e are clear losers--in general, they are off topic.

I really dislike (B), because to me the last sentence is saying that you could resolve the paradox by deciding the conclusion doesn't follow from the premises (very last part). That directly contradicts the last sentence instead of just being convoluted.

To me, that leaves (A). I will pick gibberish over something I can disprove all day long, and that's worth considering as a tactic.

The reason (A) is correct is that the stimulus tells you that your intuition gives you mixed messages--it tells you that the conclusion is both false and true and that the premises are true (the deceptive wording "follows logically from true premises" means that the conclusion and premises are true). The last sentence of the stimulus claims you have to accept that one of those intuitions is incorrect.
 Dianapoo
  • Posts: 24
  • Joined: Sep 15, 2018
|
#59211
Brook Miscoski wrote:Janie,

The reason (A) is correct is that the stimulus tells you that your intuition gives you mixed messages--it tells you that the conclusion is both false and true and that the premises are true (the deceptive wording "follows logically from true premises" means that the conclusion and premises are true). The last sentence of the stimulus claims you have to accept that one of those intuitions is incorrect.
Let me see if I can explain this in a little more detail.

CONCERNING ANY PARADOX:
"Your intuitions tell you that the conclusion of a philosophical paradox is false."
Hm, now lets read: "Solving the paradox would require you to accept that its conclusion is true."
Compare the two quotes above - If we accept that it's conclusion is true, we are (A) accepting something that intuitively seems to be incorrect.

So far, (A) holds water.

CONCERNING ANY PARADOX:
(1) "They tell you that its conclusion follows logically from true premises."

This is stating that the premises truly lead you to the conclusion for any paradox.

Hm, now lets read: (2) "Solving the paradox would require you to accept at least one of its premises is not true, or that its conclusion does not really follow logically from its premises."
Compare the two quote above. If we accept that at least one of its premises is not true, this contradicts (1), and so if we accept (2), we are accepting something that intuitively seems to be incorrect. Similarly, if we accept that its conclusion does not really follow logically from its premises, that also goes against (1) and we have to accept something that intuitively seems to be incorrect.


**I really don't like the way that they phrased (1). It should have read "your intuitions tell you that the conclusion is true as it can be derived correctly from the premises."
 James Finch
PowerScore Staff
  • PowerScore Staff
  • Posts: 943
  • Joined: Sep 06, 2017
|
#59325
Hi Diana,

As Brook noted above, this is one Must Be True question that on an actual exam is much more easily approached by process of elimination of wrong answer choices than attempting to validate a single correct answer choice, given the time constraints and placement within the section. Only (A) can survive a quick invalidation, as (C) and (D) are not addressed by the stimulus and (E) and (B) are falsified by the last sentence of it. (A) is essentially an easy inference from the second sentence in the stimulus, except for how obtuse the wording is in both the stimulus and the answer choice.

Let's look at that second sentence: "Your intuitions tell you that the conclusion of a philosophical paradox is false, but they also tell you that its conclusion follows logically from true premises," implying that you would intuit the premises to be true, as well as the conclusion to be false, despite the logic seeming to be sound. All three of those things are based on intuition, and to solve the paradox requires accepting at least one of them to be false, so one must accept something as true that intuitively seems false, which is exactly what (A) says.
 NotSureWhy
  • Posts: 8
  • Joined: Jun 08, 2019
|
#68259
Quick question: I'm a little stuck on (E). The last sentence of the stimulus says "solving a philosophical paradox requires accepting any one of three things: that its conclusion is true..." I wrote this out as: sp > ct (accepting the conclusion as true is a necessary condition for solving the paradox; all solved paradoxes have conclusions accepted as true). (E) says "If it's not possible to accept that the conclusion of a particular philosophical paradox is true, then it is not possible to solve that paradox," which I then translated as the contrapositive: -ct > -sp. Did I screw something up here? I'm NotSureWhy (see what I did there :)), but I'm just not seeing it.

Additionally, a few posts here said they eliminated this answer rather quickly? What gave it away? Is it that the impossibility of accepting is different from the truth or falsity of the conclusion?

Thanks in advance,
NotSureWhy
 Claire Horan
PowerScore Staff
  • PowerScore Staff
  • Posts: 408
  • Joined: Apr 18, 2016
|
#68426
Hi NotSureWhy (what a great username :lol: )

The stimulus sets out a necessary condition that consists of three alternatives. It says that ONE of the three is required. So it's not appropriate to extract only one of the alternatives as if it alone is necessary. You went wrong by assuming that EACH of the three was required, rather than "any of the three".

Diagramming may not be the easiest way to solve this question (see previous thread posts), but since you asked, you would have to write it as:

sp :arrow: act OR aopnt OR acdnfl

You may remember that when you take the contrapositive of a statement involving "and" or "or", you switch "and" to "or" and "or" to "and". So we end up with:

act AND aopnt AND acdnfl :arrow: sp

This makes sense if you think about it. If each of the three possibilities is absent, you cannot solve the paradox.

Great question, and good luck!

Get the most out of your LSAT Prep Plus subscription.

Analyze and track your performance with our Testing and Analytics Package.